The author's attitude toward arguments that might be based on the evidence of the rock mentioned in the passage as be...

Shula on September 13 at 12:36AM

Why not A?

Could someone plz explain why A is incorrect? I think A perfectly illustrates the author's attitude if we continue reading the lines following line 46.

Reply
Create a free account to read and take part in forum discussions.

Already have an account? log in

Emil-Kunkin on September 15 at 09:52PM

Hi, I don't think we have any support that the idea that it's difficult to believe. However, we have direct support for the idea that we need additional evidence.